USAJMO 2021

JMO 1. Let \(\mathbb{N}\) denote the set of positive integers. Find all functions \(f:\mathbb{N}\rightarrow\mathbb{N}\) such that for all positive integers \(a\) and \(b\),

\[ f(a^2+b^2)=f(a)\cdot f(b) \qquad \text{and}\qquad f(a^2)=f(a)^2 \]

Solution:

First, we have

\[ f(1^2)=f(1)^2\Longrightarrow f(1)^2-f(1)=f(1)(f(1)-1)=0 \]

Since \(f(1)\) is a positive integer, \(f(1)=1\).

\[ f(2)=f(1^2+1^2)=f(1)\cdot f(1)=1 \]

We could make a conjecture that \(f(n)=1\) for all \(n\in\mathbb{N}\). Next we will prove it by induction. Notice that \((a^2+b^2)^2=(a^2-b^2)^2+(2ab)^2\), then:

\[ \begin{aligned} f((a^2+b^2)^2)=f((a^2-b^2)^2+(2ab)^2)=f(a^2-b^2)\cdot f(2ab)\\ f((a^2+b^2)^2)=f(a^2+b^2)^2=(f(a)\cdot f(b))^2=f(a)^2\cdot f(b)^2\\ \Longrightarrow f(a^2-b^2)\cdot f(2ab)=f(a)^2\cdot f(b)^2 \qquad(*) \end{aligned} \]

If for all positive integers \(x<n,f(x)=1\). Then

  • If \(n=2k+1,k\in\mathbb{N}\). Take \(a=k+1,b=k\) in the equation \((*)\),
\[ \begin{aligned} f((k+1)^2-k^2)\cdot f(2(k+1)k)=f(2k+1)\cdot f(2k^2+2k)=f(k+1)^2\cdot f(k)^2 \end{aligned} \]

Since \(k<n\) and \(k+1<n\) ,

\[ \begin{aligned} \Longrightarrow f(k+1)^2\cdot f(k)^2=1\\ \Longrightarrow f(2k+1)\cdot f(2k^2+2k)=1 \end{aligned} \]

By \(f(x)\ge1\), it is clear that \(f(n)=f(2k+1)=1\).

  • If \(n=2k,k\in\mathbb{N}\). Take \(a=k,b=1\) in the equation \((*)\),
\[ \begin{aligned} f(a^2-b^2)\cdot f(2ab)=f(k^2-1)\cdot f(2k)=f(k)^2\cdot f(1)^2=1\\ \Longrightarrow f(n)=f(2k)=1 \end{aligned} \]

Thus, by induction, \(f(n)=1\) for all \(n\in\mathbb{N}\).

JMO 2. Rectangles \(BCC_1B_2,CAA_1C_2,\) and \(ABB_1A_2\) are erected outside an acute triangle \(ABC\). Suppose that

\[ \angle{BC_1C}+\angle{CA_1A}+\angle{AB_1B}=180^\circ \]

Prove that lines \(B_1C_2,C_1A_2\) and \(A_1B_2\) are concurrent.

Solution:

Draw the circumcircles of the three rectangles. By some simple angle chasing the original conditon implies that the circles concur at a point \(P\). Then \(\angle{CPB_2}+\angle{CPA_1}=\angle{CBB_2}+\angle{CAA_1}=180^\circ\). Hence \(P\) lies on \(A_1B_2\) etc.

\(\square\)

JMO 3.An equilateral triangle \(\Delta\) of side length \(L>0\) is given. Suppose that \(n\) equilateral triangles with side length 1 and with non-overlapping interiors are drawn inside \(\Delta\), such that each unit equilateral triangle has sides parallel to \(\Delta\), but with opposite orientation.

\[ n \leq \frac{2}{3} L^{2} \]

Unfortunately I didn't solve this one during the contest. The main idea is finding the relation between this inequality and the area of all \(\Delta\) . We could do this by drawing a regular hexagon of side length \(1/2\) above every \(\Delta\) , then prove all the hexagons are disjoint by doing casework. (the approach of Andrew Gu)

JMO 4. Carina has three pins, labeled \(A, B\), and \(C\), respectively, located at the origin of the coordinate plane. In a move, Carina may move a pin to an adjacent lattice point at distance \(1\) away. What is the least number of moves that Carina can make in order for triangle \(ABC\) to have area \(2021\)?

(A lattice point is a point \((x, y)\) in the coordinate plane where \(x\) and \(y\) are both integers, not necessarily positive.)

Notice that moving \(ABC\) by the same vector won't change its area. Denote \(A(x_A,y_A),B(x_B,y_B),C(x_C,y_C)\). Let \(n\) be the number of moves. When \(n\) is minimum, we have:

\[ \sum_{cyc}(|x_A|+|y_A|)\le\sum_{cyc}(|x_A-1|+|y_A|) \]

Which means there are more non-negative \(x\) than negative ones. Similarly, there are more non-positive \(x\) than positive ones. Thus, at least one \(x\) coordinate is \(0\). Also at least one \(y\) coordinate is \(0\).

  • If \(A(0,0)\) ,
\[ \begin{aligned} 2S_\triangle{ABC}&=|x_By_C-x_Cy_B|\\ &\le |x_B||y_C|+|x_C||y_B|\\ &\le \frac{1}{4}((|x_B|+|y_C|)^2+(|x_C|+|y_B|^2))\\ &\le \frac{1}{4}n^2\\ \end{aligned} \]

Thus, \(\dfrac{1}{4}n^2\ge2\cdot2021=4042\), which means \(n\ge128\).

  • If \(A(x_A,0),B(0,y_B)\) , WLOG \(x_A,y_B\ge 0, x_C,y_C\le 0\) .
\[ \begin{aligned} 2S_\triangle{ABC}&=|x_Ay_C+x_Cy_B+x_Cy_C|\\ &= (x_A+|x_C|)(y_B+|y_C|)-x_Ay_B\\ &\le \frac{1}{4}(x_A+|x_C|+y_B+|y_C|)^2\\ &\le \frac{1}{4}n^2\\ \end{aligned} \]

This also leads to \(n\ge 128\).

When \(n=128\), \(A(58,0),B(0,55),C(-6,-9)\) has area \(2021\) .

Hence the answer is \(\boxed{128}\) .

JMO 5. A finite set \(S\) of positive integers has the property that, for each \(s \in S,\) and each positive integer divisor \(d\) of \(s\), there exists a unique element \(t \in S\) satisfying \(\text{gcd}(s, t) = d\). (The elements \(s\) and \(t\) could be equal.)

Given this information, find all possible values for the number of elements of \(S\).

Solution:

The answer is \(0,1\) and power of \(2\). \(S=\varnothing,\{1\}\) satisfy the condition obviously. Next, we want to show that for any \(|S|\ge2\) , \(|S|=2^k\).

1) For \(|S|=2^k\) , pick \(2k\) distinct prime numbers and denote them as \(a_1,a_2,\dots,a_k\) and \(b_1,b_2,\dots,b_k\). Let \(S\) be the set of all \(s=\prod_{i=1}^k p_i\) , \(p_i=a_i\) or \(p_i=b_i\) for each \(i\). For each \(s\in S\), WLOG \(s=\prod_{i=1}^k a_i\) ( \(b_i\) is another possible value of \(p_i\) ). For

\[ \gcd(s,t)=d=\prod_{i=1}^{m}a_{x_i} \]

For every position \(x_i\) , \(p_{x_i}=a_{x_i}\) for \(t\) . And for the other positions \(i\), \(p_i=b_i\). \(t\in S\) is clearly unique for every positive divisor \(d\) .

Lemma: Let \(D_s\) denote the set of positive divisors \(s\) . For \(s\in S\), \(|D_s|=|S|\).

Proof: Since there exist a unique \(t\in S\) for every \(d\in D_s\) that \(\gcd(s,t)=d\), and for every \(t\in S\) we also have \(\gcd(s,t)\in D_s\). This creates a bijection from \(S\) to \(D_s\), which means \(|D_s|=|S|\).

\(\square\)

2) \(|S|\neq2^k\) that satisfies the condition doesn't exist.

Let \(s=\prod_{i=1}^k p_i^{\alpha_i}\in S\), then

\[ |D_s|=\prod_{i=1}^k (\alpha_i+1)=|S| \]

There exist \(\alpha_i>1\) or else \(|S|\) will be a power of \(2\). WLOG, \(\alpha_1>1\), then let

\[ \begin{aligned} \gcd(s,t)=d &=p_1^{\alpha_1-1}\prod_{i=2}^k p_i^{\alpha_i}\\ \end{aligned} \]

Since \(\gcd(s,\dfrac td)=1\) and consider \(|D_s|\) as a multiplicative function about \(s\), \(|D_t|\) is a multiple of \(|D_d|=\alpha_1\prod_{i=2}^k(\alpha_i+1)\) . However, notice that since \(t\in S\), \(|D_t|=|S|=\prod_{i=1}^k (\alpha_i+1)\) . This means

\[ \alpha_1|\alpha_1+1\Longrightarrow \alpha_1=1 \]

Contradiction!

Thus, the only possible value is \(|S|=2^k\).

JMO 6. Let \(n \ge 4\) be an integer. Find all positive real solutions to the following system of \(2n\) equations:

\[ \begin{aligned} a_{1} &=\frac{1}{a_{2 n}}+\frac{1}{a_{2}}, & a_{2}&=a_{1}+a_{3}, \\ a_{3}&=\frac{1}{a_{2}}+\frac{1}{a_{4}}, & a_{4}&=a_{3}+a_{5}, \\ a_{5}&=\frac{1}{a_{4}}+\frac{1}{a_{6}}, & a_{6}&=a_{5}+a_{7} \\ &\vdots & &\vdots \\ a_{2 n-1}&=\frac{1}{a_{2 n-2}}+\frac{1}{a_{2 n}}, & a_{2 n}&=a_{2 n-1}+a_{1} \end{aligned} \]

Solution:

The answer is that the only solution is \(a_{2i-1}=1,a_{2i}=2\) for \(i=1,2,\dots,n\) which works.

  • \(n=2k+1\), if there exists \(a_{2i-1},a_{2i+1}\ge1\) for some positive integer \(i\) , WLOG \(a_1,a_3\ge1\) . Then
\[ \begin{aligned} a_2&=a_1+a_3\ge2\\ \frac1{a_4}&=a_3-\frac1{a_2}\ge 1-\frac12=\frac12\Longrightarrow a_4\le2\\ a_5 &=a_4-a_3\le 2-1=1\\ \frac1{a_6}&=a_5-\frac1{a_4}\le 1-\frac12=\frac12\Longrightarrow a_6\ge2\\ a_7&=a_6-a_5\ge2-1=1\\ &\vdots \end{aligned} \]

By this process we could get \(a_{4i}\le2,a_{4i+1}\le1,a_{4i+2}\ge2,a_{4i+3}\ge1\) for indices \(\pmod{2n}\). With \(2n=4k+2\), we would get for every even indices \(2\le a_i\le2\), and for every odd indices \(1\le a_i\le 1\). Hence it's the only solution in this case.

  • It is similar when we have \(a_{2i-1},a_{2i+1}\le 1\) . Notice that
\[ \prod_{i=1}^n (a_{2i-1}-1)(a_{2i+1}-1)=\prod_{i=1}^n(a_{2i-1}-1)^2\ge 0 \]

And \(n\) is odd, so there's always \(i\) that satisfies \((a_{2i-1}-1)(a_{2i+1}-1)\ge 0\).

  • \(n=2k\) , if there exists \(a_{2i-1},a_{2i+1}\ge1\) for some positive integer \(i\) , WLOG \(a_1,a_3\ge1\) . Then we will have a process similar to the first case.

By this process we could get \(a_{4i}\le2,a_{4i+1}\le1,a_{4i+2}\ge2,a_{4i+3}\ge1\) for indices \(\pmod{2n}\). Hence \(a_1=1\) and notice that \(a_{2n}=a_1+a_{2n-1}\ge2\), only \(a_{2n}=2,a_{2n-1}=1\) satisfies it. By knowing those three values it's sufficient to see that \(a_{2i-1}=1,a_{2i}=2\) for \(i=1,2,\dots,n\).

  • If there doesn't exist \((a_{2i-1}-1)(a_{2i+1}-1)\ge 0\) , we will have that
\[ \begin{aligned} a_1\ge1\ge a_3\le1\le a_5\ge1\ge a_7\cdots\\ \Longrightarrow a_1=\frac1{a_{2n}}+\frac1{a_2}\ge a_3=\frac1{a_{2}}+\frac1{a_4}\\ \Longrightarrow \frac1{a_{2n}}\ge\frac1{a_4}\\ a_5\ge a_7\Longrightarrow \frac1{a_4}\ge\frac{1}{a_8}\\ \vdots\\ a_{4k-3}\ge a_{4k-1}\Longrightarrow \frac1{a_{4k-4}}\ge\frac1{a_{4k}}\\ \Longrightarrow a_4=a_8=\cdots=a_{2n}\\ \end{aligned} \]

Since

\[ \begin{aligned} a_1=\frac{1}{a_{2n}}+\frac{1}{a_2}=\frac1{a_4}+\frac{1}{a_2}=a_3\\ a_1\ge1\ge a_3\Longrightarrow a_1=a_3=1\\ a_2=a_1+a_3=2,a_4=2\\ \end{aligned} \]

Then it's sufficient to see that \(a_{2i-1}=1,a_{2i}=2\) for all integer \(1\le i\le n\) .

Comments